LSAT and Law School Admissions Forum

Get expert LSAT preparation and law school admissions advice from PowerScore Test Preparation.

 Adam Tyson
PowerScore Staff
  • PowerScore Staff
  • Posts: 5153
  • Joined: Apr 14, 2011
|
#49576
Complete Question Explanation

See the Setup and Rules diagram thread discussion here: lsat/viewtopic.php?f=56&t=11790.

The correct answer choice is (E).

The way to attack this game IS mostly through brute force, albeit with a few hypotheticals at the outset to get a sense of the game and, we hope, make a couple of inferences. One of those inferences should have been that no matter who goes last, he get W (and so L CAN get W by going last, so B is out).

Answer choice (A): To the extent that any hypotheticals you did up to this point, either in the setup or for prior questions, enables you to knock out some of these answers, then knock them out first. The rest, you may have to test. So, assuming that you haven't knocked out anything other than B at this stage, let's test the rest, starting with answer A by seeing how we might get J to select X:

J would have to go at a time when Y was gone but X was still around. So what if P went first, taking Y, and then J went second? Bingo - J CAN get X, so A is wrong.

Answer choice (B): B is already out, so let's try C:

Answer choice (C): For L to get Z, X has to be gone but Z still must be available. What if T went first, taking X, and L went second? Boom - L CAN get Z, so C is out.

Answer choice (D): For D, how might we get T to get X? See answer C! T goes first, and shazam, he gets X. That's out.

Answer choice (E): This is the correct answer choice. Now, to test E, how would P get X? None of my hypos and solutions have shown that, so I am liking it so far. Plus, I have eliminated everything else, so I might skip testing and just pick it. But let's work it through anyway, if only to get a better understanding of the game in this untimed practice. To get X, P would have to go third, after Y and Z are gone (and remember, if he went last he would get W, a global inference). So who will take Y and Z in the first two turns? L and T would both pick X first, so they can't go before P, because then they would take X and P would not get it...

Wait a minute, if neither L nor T goes before P, then P must go first or second, and he will definitely get either his first choice (Y) or else his second choice (Z)! Turns out there is no way for P to select X! It will be gone before he gets his turn, or else he will select something that he likes better! We have our winner.

Pattern games like this do indeed involve a lot of brute force, sadly. The plus sides are that the setup won't usually take long, and the other three games in the section will likely be a lot easier and faster.

Get the most out of your LSAT Prep Plus subscription.

Analyze and track your performance with our Testing and Analytics Package.